Difference between revisions of "2012 USAJMO Problems/Problem 5"

m (fixing bad LaTeX)
Line 3: Line 3:
 
For distinct positive integers <math>a</math>, <math>b < 2012</math>, define <math>f(a,b)</math> to be the number of integers <math>k</math> with <math>1 \le k < 2012</math> such that the remainder when <math>ak</math> divided by 2012 is greater than that of <math>bk</math> divided by 2012.  Let <math>S</math> be the minimum value of <math>f(a,b)</math>, where <math>a</math> and <math>b</math> range over all pairs of distinct positive integers less than 2012.  Determine <math>S</math>.
 
For distinct positive integers <math>a</math>, <math>b < 2012</math>, define <math>f(a,b)</math> to be the number of integers <math>k</math> with <math>1 \le k < 2012</math> such that the remainder when <math>ak</math> divided by 2012 is greater than that of <math>bk</math> divided by 2012.  Let <math>S</math> be the minimum value of <math>f(a,b)</math>, where <math>a</math> and <math>b</math> range over all pairs of distinct positive integers less than 2012.  Determine <math>S</math>.
  
== Solution ==
+
== Solutions ==
 +
 
 +
=== Solution 1 ===
 +
 
 +
Let <math>ak \equiv r_{a} \pmod{2012}</math> and <math>bk \equiv r_{b} \pmod{2012}</math>. Notice that this means <math>a(2012 - k) \equiv 2012 - r_{a} \pmod{2012}</math> and <math>b(2012 - k) \equiv 2012 - r_{b} \pmod{2012}</math>. Thus, for every case where <math>r_{a} > r_{b}</math>, there is a case where <math>r_{b} > r_{a}</math>. Therefore, we merely have to calculate <math>\frac{1}{2}</math> times the number of values of <math>k</math> for which <math>r_{a} \neq r_{b}</math> and <math>r_{a} \neq 0</math>.
 +
 
 +
 
 +
However, the answer is NOT <math>\frac{1}{2}(2012) = 1006</math>! This is because we must count the cases where the value of <math>k</math> makes <math>r_{a} = r_{b}</math> or where <math>r_{a} = 0</math>.
 +
 
 +
 
 +
So, let's start counting.
 +
 
 +
 
 +
If <math>k</math> is even, we have either <math>a \equiv 0 \pmod{1006}</math> or <math>a - b \equiv 0 \pmod{1006}</math>. So, <math>a = 1006</math> or <math>a = b + 1006</math>. We have <math>1005</math> even values of <math>k</math> (which is all the possible even values of k, since the two above requirements don't require <math>k</math> at all).
 +
 
 +
 
 +
If <math>k</math> is odd, if <math>k = 503</math> or <math>k = 503 \cdot 3</math>, then <math>a \equiv 0 \pmod{4}</math> or <math>a \equiv b \pmod{4}</math>. Otherwise, <math>ak \equiv 0 \pmod{2012}</math> or <math>ak \equiv bk \pmod{2012}</math>, which is impossible to satisfy, given the domain <math>a, b < 2012</math>. So, we have <math>2</math> values of <math>k</math>.
 +
 
 +
 
 +
In total, we have <math>2 + 1005 = 1007</math> values of <math>k</math> which makes <math>r_{a} = r_{b}</math> or <math>r_{a} = 0</math>, so there are <math>2011 - 1007 = 1004</math> values of <math>k</math> for which <math>r_{a} \neq r_{b}</math> and <math>r_{a} \neq 0</math>. Thus, by our reasoning above, our solution is <math>\frac{1}{2} \cdot 1004 = \boxed{502}</math>.
 +
 
 +
 
 +
Solution by <math>\textbf{\underline{Invoker}}</math>
 +
 
 +
 
 +
=== Solution 2 ===
 
The key insight in this problem is noticing that when <math>ak</math> is higher than <math>bk</math>, <math>a(2012-k)</math> is lower than <math> b(2012-k)</math>, except at <math>2 \pmod{4}</math> residues*. Also, they must be equal many times. <math>2012=2^2*503</math>. We should have multiples of <math>503</math>. After trying all three pairs and getting <math>503</math> as our answer, we win. But look at the <math>2\pmod{4}</math> idea. What if we just took <math>2</math> and plugged it in with <math>1006</math>?
 
The key insight in this problem is noticing that when <math>ak</math> is higher than <math>bk</math>, <math>a(2012-k)</math> is lower than <math> b(2012-k)</math>, except at <math>2 \pmod{4}</math> residues*. Also, they must be equal many times. <math>2012=2^2*503</math>. We should have multiples of <math>503</math>. After trying all three pairs and getting <math>503</math> as our answer, we win. But look at the <math>2\pmod{4}</math> idea. What if we just took <math>2</math> and plugged it in with <math>1006</math>?
 
We get <math>502</math>.
 
We get <math>502</math>.
 
  --[[User:Va2010|Va2010]] 11:12, 28 April 2012 (EDT)va2010
 
  --[[User:Va2010|Va2010]] 11:12, 28 April 2012 (EDT)va2010
  
== Alternate Solution==
+
=== Solution 3 ===
 
Say that the problem is a race track with <math>2012</math> spots. To intersect the most, we should get next to each other a lot so the negation is high. As <math>2012=2^2*503</math>, we intersect at a lot of multiples of <math>503</math>.
 
Say that the problem is a race track with <math>2012</math> spots. To intersect the most, we should get next to each other a lot so the negation is high. As <math>2012=2^2*503</math>, we intersect at a lot of multiples of <math>503</math>.
  

Revision as of 19:41, 23 February 2019

Problem

For distinct positive integers $a$, $b < 2012$, define $f(a,b)$ to be the number of integers $k$ with $1 \le k < 2012$ such that the remainder when $ak$ divided by 2012 is greater than that of $bk$ divided by 2012. Let $S$ be the minimum value of $f(a,b)$, where $a$ and $b$ range over all pairs of distinct positive integers less than 2012. Determine $S$.

Solutions

Solution 1

Let $ak \equiv r_{a} \pmod{2012}$ and $bk \equiv r_{b} \pmod{2012}$. Notice that this means $a(2012 - k) \equiv 2012 - r_{a} \pmod{2012}$ and $b(2012 - k) \equiv 2012 - r_{b} \pmod{2012}$. Thus, for every case where $r_{a} > r_{b}$, there is a case where $r_{b} > r_{a}$. Therefore, we merely have to calculate $\frac{1}{2}$ times the number of values of $k$ for which $r_{a} \neq r_{b}$ and $r_{a} \neq 0$.


However, the answer is NOT $\frac{1}{2}(2012) = 1006$! This is because we must count the cases where the value of $k$ makes $r_{a} = r_{b}$ or where $r_{a} = 0$.


So, let's start counting.


If $k$ is even, we have either $a \equiv 0 \pmod{1006}$ or $a - b \equiv 0 \pmod{1006}$. So, $a = 1006$ or $a = b + 1006$. We have $1005$ even values of $k$ (which is all the possible even values of k, since the two above requirements don't require $k$ at all).


If $k$ is odd, if $k = 503$ or $k = 503 \cdot 3$, then $a \equiv 0 \pmod{4}$ or $a \equiv b \pmod{4}$. Otherwise, $ak \equiv 0 \pmod{2012}$ or $ak \equiv bk \pmod{2012}$, which is impossible to satisfy, given the domain $a, b < 2012$. So, we have $2$ values of $k$.


In total, we have $2 + 1005 = 1007$ values of $k$ which makes $r_{a} = r_{b}$ or $r_{a} = 0$, so there are $2011 - 1007 = 1004$ values of $k$ for which $r_{a} \neq r_{b}$ and $r_{a} \neq 0$. Thus, by our reasoning above, our solution is $\frac{1}{2} \cdot 1004 = \boxed{502}$.


Solution by $\textbf{\underline{Invoker}}$


Solution 2

The key insight in this problem is noticing that when $ak$ is higher than $bk$, $a(2012-k)$ is lower than $b(2012-k)$, except at $2 \pmod{4}$ residues*. Also, they must be equal many times. $2012=2^2*503$. We should have multiples of $503$. After trying all three pairs and getting $503$ as our answer, we win. But look at the $2\pmod{4}$ idea. What if we just took $2$ and plugged it in with $1006$? We get $502$.

--Va2010 11:12, 28 April 2012 (EDT)va2010

Solution 3

Say that the problem is a race track with $2012$ spots. To intersect the most, we should get next to each other a lot so the negation is high. As $2012=2^2*503$, we intersect at a lot of multiples of $503$.

See also

2012 USAJMO (ProblemsResources)
Preceded by
Problem 4
Followed by
Problem 6
1 2 3 4 5 6
All USAJMO Problems and Solutions

The problems on this page are copyrighted by the Mathematical Association of America's American Mathematics Competitions. AMC logo.png